LSAT and Law School Admissions Forum

Get expert LSAT preparation and law school admissions advice from PowerScore Test Preparation.

 Jon Denning
PowerScore Staff
  • PowerScore Staff
  • Posts: 904
  • Joined: Apr 11, 2011
|
#37221
Question #2 is a Global Could Be True, meaning a single choice is possible, while the other four cannot occur.

In much the same way that we approached the first question (a Global List), let's try to eliminate the four wrong answer choices, as those are the absolutes of this set and thus should be easier to identify.

Answer choice (A): This cannot occur because if J is immediately after F we would have a KFJ block, and this game doesn't have three consecutive spaces open since G is in the middle in 3. Put another way, F is always either in 2 (immediately ahead of G), or in 5 (with nothing after it).

Answer choice (B): This is the correct answer choice. G is always in 3, so what this answer is suggesting by putting H immediately after G is that H could go in 4. That is an entirely acceptable arrangement, and would give us this order:

..... K F G H J

If I was approaching this question on the actual exam and saw B I would note that it doesn't appear to cause a problem, keep it, and move on to the remaining three looking for issues in each. Let's do that.

Answer choice (C): Much like answer choice A, this creates a three-piece block that cannot exist: HKF. There simply isn't room with G in 3. Also like A, you could view this as impossible based on K's placement in either spot 1 (nothing ahead of it), or in spot 4 (G immediately ahead of it).

Answer choice (D): This is functionally a duplicate of answer choice C and is wrong for the exact same reasons.

Answer choice (E): We know from the second rule that F comes immediately after K, so a KG block is impossible.

Get the most out of your LSAT Prep Plus subscription.

Analyze and track your performance with our Testing and Analytics Package.